Stability of circular orbits in an arbitrary central force field

In summary, the stability of an object orbiting in a circular orbit of radius r_c in an arbitrary force field f is considered. The author arrives at the equation of a harmonic oscillator for small deviations x from the circular orbit. The author claims that in order to prove instability for the case where f = -\frac{c}{r^3}, the second order term must be kept when approximating the equation. However, the author's mention of using the second order terms may be misleading as it is not necessary for proving instability in this case. By setting l such that Veff′(rc)=0, it can be shown that circular orbits are unstable for inverse-cube forces.
  • #1
pop_ianosd
13
2
TL;DR Summary
I'm trying to understand why a stability criterion derived in a course for circular orbits in a general force field is valid.
I'm also trying to obtain a simple argument for why circular orbits in an inverse-cubed force field are unstable.
In this chapter, the stability of an object orbiting in a circular orbit of radius [itex]r_c[/itex] in an arbitrary force field [itex]f[/itex] is considered.
The author arrives at the equation of a harmonic oscillator, for small deviations [itex]x[/itex] from the circular orbit:
[tex] \ddot{x} + \left[-3\frac{f(r_c)}{r_c} - f'(r_c)\right]x= 0 [/tex]
From here follows that if the sign of the coefficient of [itex]x[/itex] is positive the orbit is stable.
My most specific question is about the case where [itex]f = -\frac{c}{r^3}[/itex]. The author claims that in order to prove that it is unstable, we must keep the second order term when approximating the equation (308). That equation, with such a specific force would look like this:
[tex] \ddot{x} = \frac{h^2 - c}{(r_c + x)^3} [/tex]
Since we are considering circular orbits, we have [itex]-\frac{h^2}{r_c^3} = -\frac{c}{r_c^3}[/itex] (eq. 306 in the link), so the right term in the equation above is always zero, and I therefore don't see how keeping the second order term would help.
My question is: what am I doing wrong, or do you have a better guess for what the author meant.
The way I would prove instability in this case is to derive the solutions to the unapproximated equation, and observe that the initial conditions that produce bounded solutions sit on a line, and not in a volume, in the initial conditions space. But I'm not too satisfied with this argument, as it deals with the concept of boundness rather than stability, and it just doesn't seem very elegant.

Returning to the criterion above, I'm also not convinced of it. I have the intuition that it would be a necessary condition for stability, but I wonder whether it is sufficient.
 
Physics news on Phys.org
  • #2
pop_ianosd said:
how keeping the second order term would help
it shows there is no restoring force: ##\ \ddot x = 0##
 
  • #3
As usual, it's much simpler using the Lagrange formalism. You have
$$L=\frac{m}{2} \dot{\vec{r}}^2 - V(r).$$
From rotational invariance you have angular-momentum conservation, and thus the orbit is in a plane perpendicular to it. This reduces the problem to a 1D effective motion for ##r##:
$$L_{\text{eff}}=\frac{m}{2} \dot{r}^2-V_{\text{eff}}(r)$$
with
$$V_{\text{eff}}(r)=V(r)+\frac{l^2}{2m r^2}.$$
For the solution ##r=r_c=\text{const}## to be stable the effective potential must have a minimum at ##r=r_c##. A sufficient condition is ##V_{\text{eff}}'(r_c)=0## and ##m\omega^2:=V_{\text{eff}}''(r_c)>0##.

The setting ##r=r_c+x## the equation of motion for small ##x## can be approximated as the motion in the potential
$$\tilde{V}(x)=\frac{m}{2} \omega^2 x^2,$$
leading to the harmonic-oscillator equation
$$\ddot{x}=-\omega^2 x.$$
Since ##\omega^2>0##, this admits solutions for ##x## staying small for all time, and that's what makes the circular orbit small.

If ##V_{\text{eff}}## has a maximum at ##r=r_c## the circular orbit is for sure unstable since then
$$\tilde{V}(x)=-\frac{m}{2} \lambda^2 x^2, \quad \lambda^2>0$$
and the EoM for ##x## is approximately (close to ##x=0##)
$$\ddot{x}=+\lambda^2 x\; \Rightarrow \; x(t)=A \exp(\lambda t) +B \exp(-\lambda t),$$
i.e., you always find solutions exponentially growing, i.e., the circular orbit is not stable under arbitrary small perturbations, and the approximation breaks rapidly down.

For ##V_{\text{eff}}''(r_c)=0## you need to go to higher order in the series expansion of ##V_{\text{eff}}## in order to decide about the stability.
 
  • Like
Likes BvU
  • #4
Thank you for the answers!
The image of the convex effective potential gives a nice intuitive reason for why the condition is sufficient for stability.
I'm not sure I understand what you mean by
vanhees71 said:
The setting r=rc+x the equation of motion for small x can be approximated as the motion in the potential
V~(x)=m2ω2x2,
leading to the harmonic-oscillator equation
x¨=−ω2x.
Since ω2>0, this admits solutions for x staying small for all time, and that's what makes the circular orbit small.
Is this meant to be an argument for why the orbit is stable? Because if that's the case my problem with it is that I don't see how an approximation can prove something about the initial problem.

In the case of an inverse-cube force, you will then have a constant effective potential , if you set l such that Veff′(rc)=0. So then I guess the analysis is even simpler, in that the orbiting object is simply free to move in terms of r. Basically what @BvU said about there being no force.

Would it then be safe to conclude that the mention of using the second order terms is misleading?
The case
img862.png
is special, since the first-order terms in the expansion of Equation (308) cancel out exactly, and it is necessary to retain the second-order terms. Doing this, it is easily demonstrated that circular orbits are also unstable for inverse-cube (
img862.png
) forces.
 

1. What is a central force field?

A central force field is a type of force field where the force acting on an object is always directed towards a single point, known as the center of force. Examples of central force fields include gravity and electrostatic force.

2. What does it mean for an orbit to be stable?

An orbit is considered stable if the object moving in the orbit is able to maintain a relatively constant distance from the center of force and does not deviate significantly from its original path. This means that the object will continue to orbit around the center of force without collapsing into it or flying off into space.

3. How is stability of circular orbits determined in an arbitrary central force field?

The stability of circular orbits in an arbitrary central force field can be determined by analyzing the potential energy and kinetic energy of the object in the orbit. If the potential energy is greater than the kinetic energy, the orbit will be unstable and the object will either collapse or fly off. If the kinetic energy is greater than the potential energy, the orbit will be stable.

4. What factors affect the stability of circular orbits in an arbitrary central force field?

The stability of circular orbits can be affected by various factors, such as the strength of the central force, the mass of the object, and the distance between the object and the center of force. Additionally, the shape of the central force field and the presence of other objects in the system can also impact the stability of an orbit.

5. Can circular orbits be stable in all types of central force fields?

No, circular orbits cannot be stable in all types of central force fields. For example, in a central force field with an inverse square law, such as gravity, circular orbits are only stable if the central force is attractive. If the central force is repulsive, the orbit will not be stable and the object will either spiral away or crash into the center of force.

Similar threads

Replies
1
Views
2K
  • Classical Physics
Replies
2
Views
482
  • Classical Physics
Replies
7
Views
9K
Replies
1
Views
576
  • Classical Physics
Replies
17
Views
1K
  • Classical Physics
Replies
0
Views
149
Replies
3
Views
498
  • Advanced Physics Homework Help
Replies
3
Views
1K
  • Advanced Physics Homework Help
Replies
26
Views
3K
  • Advanced Physics Homework Help
Replies
11
Views
1K
Back
Top